Gull and Gill's theory

Post a reply


This question is a means of preventing automated form submissions by spambots.

BBCode is ON
[img] is ON
[flash] is OFF
[url] is ON
Smilies are OFF
Topic review
   

Expand view Topic review: Gull and Gill's theory

Re: Gull and Gill's theory

Post by FrediFizzx » Sun May 16, 2021 4:52 am

Joy Christian wrote:
FrediFizzx wrote:It doesn't look like anything was fixed yet on the paper,

https://arxiv.org/abs/2012.00719

I already showed how to fix eq.(1) by specifying actual A and B measurement functions. Do you want me to show how to fix eq. (4)??? It is pretty easy. :mrgreen:

Gill is not coming back. Perhaps karma has finally caught up with him? In the irrational world in which a non-theorem like Bell's "theorem" holds, the law of karma would hold too!
.

Actually, eq. (1) still has a problem. It's non-local.
.

Re: Gull and Gill's theory

Post by Joy Christian » Sun May 16, 2021 3:29 am

FrediFizzx wrote:It doesn't look like anything was fixed yet on the paper,

https://arxiv.org/abs/2012.00719

I already showed how to fix eq.(1) by specifying actual A and B measurement functions. Do you want me to show how to fix eq. (4)??? It is pretty easy. :mrgreen:

Gill is not coming back. Perhaps karma has finally caught up with him? In the irrational world in which a non-theorem like Bell's "theorem" holds, the law of karma would hold too!
.

Re: Gull and Gill's theory

Post by FrediFizzx » Thu May 13, 2021 5:05 pm

It doesn't look like anything was fixed yet on the paper,

https://arxiv.org/abs/2012.00719

I already showed how to fix eq.(1) by specifying actual A and B measurement functions. Do you want me to show how to fix eq. (4)??? It is pretty easy. :mrgreen:
.

Re: Gull and Gill's theory

Post by FrediFizzx » Mon May 10, 2021 5:44 pm

Justo wrote:
FrediFizzx wrote:A lot of it is pretty much self-explanatory. The rest I can easily explain to you. What is the first part you don't understand?
.

Thank you. Another question, is there some kind of free version that one can use?

You can do a 15 day free trial.

https://www.wolfram.com/mathematica/trial/
.

Re: Gull and Gill's theory

Post by Justo » Mon May 10, 2021 4:38 pm

FrediFizzx wrote:A lot of it is pretty much self-explanatory. The rest I can easily explain to you. What is the first part you don't understand?
.

Thank you. Another question, is there some kind of free version that one can use?

Re: Gull and Gill's theory

Post by FrediFizzx » Mon May 10, 2021 6:54 am

Justo wrote:
FrediFizzx wrote: Apparently you don't believe or understand our proof that shoots down Bell's theory (interpretation). If any questions or comments about our proof, please ask. I would be happy to explain it to you,
.

That's right. I can't understand because can't read your code. I am not good at computer programs. But I am curious. Please give some reference to your computer language and maybe will try to learn it whenever I find the time and then get back to you asking questions.

https://www.wolfram.com/mathematica/

A lot of it is pretty much self-explanatory. The rest I can easily explain to you. What is the first part you don't understand?
.

Re: Gull and Gill's theory

Post by Justo » Mon May 10, 2021 6:49 am

FrediFizzx wrote: Apparently you don't believe or understand our proof that shoots down Bell's theory (interpretation). If any questions or comments about our proof, please ask. I would be happy to explain it to you,
.

That's right. I can't understand because can't read your code. I am not good at computer programs. But I am curious. Please give some reference to your computer language and maybe will try to learn it whenever I find the time and then get back to you asking questions.

Re: Gull and Gill's theory

Post by Joy Christian » Mon May 10, 2021 6:06 am

Justo wrote:
However, when the Bell derivation is correctly interpreted that problem does not exist.

Anything and everything can be "correctly interpreted" to justify the conclusion we already believe in. That is why Bell's theorem is a belief system, not science.
.

Re: Gull and Gill's theory

Post by FrediFizzx » Mon May 10, 2021 6:05 am

Justo wrote:
FrediFizzx wrote:It is really hard to believe that any mathematician would believe such nonsense. The A1 in the first pair is impossible to be the same as the A1 in the second pair, etc. So, you could have,

A1 B1= +1, A1 B2= -1, A2 B1= -1, A2 B2= -1 --> +1+1+1+1 = 4

Bell was wrong, get over it and move on!!!!!
.

I agree with @FredFizzx, there is an interesting article by Adenier that I like because it explains this problem very clearly https://arxiv.org/abs/quant-ph/0006014. It was published in a Conference Proceedings Foundations of Probability and Physics, chapter Refutation of Bell’s Theorem, pages 29–38. World Scientific, 2001.

However, when the Bell derivation is correctly interpreted that problem does not exist.

Apparently you don't believe or understand our proof that shoots down Bell's theory (interpretation). If any questions or comments about our proof, please ask. I would be happy to explain it to you,

Image
Image
.

Re: Gull and Gill's theory

Post by Justo » Mon May 10, 2021 5:10 am

FrediFizzx wrote:It is really hard to believe that any mathematician would believe such nonsense. The A1 in the first pair is impossible to be the same as the A1 in the second pair, etc. So, you could have,

A1 B1= +1, A1 B2= -1, A2 B1= -1, A2 B2= -1 --> +1+1+1+1 = 4

Bell was wrong, get over it and move on!!!!!
.

I agree with @FredFizzx, there is an interesting article by Adenier that I like because it explains this problem very clearly https://arxiv.org/abs/quant-ph/0006014. It was published in a Conference Proceedings Foundations of Probability and Physics, chapter Refutation of Bell’s Theorem, pages 29–38. World Scientific, 2001.

However, when the Bell derivation is correctly interpreted that problem does not exist.

Re: Gull and Gill's theory

Post by FrediFizzx » Sun May 09, 2021 2:40 pm

gill1109 wrote:
FrediFizzx wrote:
gill1109 wrote:Sorry Fred, that was an illegitimate substitution which you just did. ...

How could it be illegitimate? Don't make nonsensical claims like that without backing it up. Are A and B equal to +1 or -1 or not? Of course they are. So their product is always going to be +1 or -1. The formulation produces nonsense just like I said. You have to figure out a way to keep the vectors a and b in the calculation while producing the +/-1's.
.
...
If you use the CHSH inequality to prove the theorem you don’t even need any calculus. Since there are only two a’s in the game and only two b’s, everything depends on a single list of 16 probabilities adding up to +1. Because (in obvious notation) there only 16 possible values of the quadruple (A1, A2, B1, B2). As lambda varies it takes values in the set {-1, +1}^4. That’s a set of 16 elements. It’s easy to show that A1 B1 - A1 B2 - A2 B1 - A2 B2 always equals -2 or +2. Its mean value therefore lies between -2 and +2 (inclusive)...

It is really hard to believe that any mathematician would believe such nonsense. The A1 in the first pair is impossible to be the same as the A1 in the second pair, etc. So, you could have,

A1 B1= +1, A1 B2= -1, A2 B1= -1, A2 B2= -1 --> +1+1+1+1 = 4

Bell was wrong, get over it and move on!!!!!
.

Re: Gull and Gill's theory

Post by FrediFizzx » Sun May 09, 2021 8:31 am

gill1109 wrote:
FrediFizzx wrote:
gill1109 wrote:Sorry Fred, that was an illegitimate substitution which you just did. ...

How could it be illegitimate? Don't make nonsensical claims like that without backing it up. Are A and B equal to +1 or -1 or not? Of course they are. So their product is always going to be +1 or -1. The formulation produces nonsense just like I said. You have to figure out a way to keep the vectors a and b in the calculation while producing the +/-1's.
.
... A and B are not identically equal to -1 or identically equal to +1. ...

Egads! More freakin' nonsense! This proves that you and Bell are wrong.

Image
Image

PlotArray1 is a partial list of the A and B outcome pairs. Note that the limit replacement function on the A and B functions give the 4 different outcome pairs.

Bell loses again! :mrgreen: :mrgreen: :mrgreen: :mrgreen:

Re: Gull and Gill's theory

Post by Joy Christian » Sun May 09, 2021 8:12 am

gill1109 wrote:
Joy Christian does not have a counterexample to Bell’s theorem … because Bell’s theorem is a simple, true, theorem of elementary pure mathematics.

Both parts of the above statement have been proven wrong many times before. For the latest demonstrations of why they are wrong, please see my latest paper, published in IEEE Access.

PS: The above paper of mine has gone through two rounds of peer-review involving 14 reviewers. That means that Gill lives in his own fantasy world and is not believed by the community.
.

Re: Gull and Gill's theory

Post by gill1109 » Sun May 09, 2021 6:49 am

FrediFizzx wrote:
gill1109 wrote:Sorry Fred, that was an illegitimate substitution which you just did. ...

How could it be illegitimate? Don't make nonsensical claims like that without backing it up. Are A and B equal to +1 or -1 or not? Of course they are. So their product is always going to be +1 or -1. The formulation produces nonsense just like I said. You have to figure out a way to keep the vectors a and b in the calculation while producing the +/-1's.
.

Dear Fred

A and B are not identically equal to -1 or identically equal to +1. For given a and b their product still depends on lambda. As lambda varies, their product can take both values. Not at the same time. But for some lambda, their product can be +1. For other lambda, their product can be -1. There are no other possibilities.

For each a, b and lambda, A(a, lambda) equals +1 or it equals -1, and B(b, lambda) equals +1 or it equals -1.

Joy Christian does not have a counterexample to Bell’s theorem … because Bell’s theorem is a simple, true, theorem of elementary pure mathematics. Which can be proved in a myriad of different ways.

If you use the CHSH inequality to prove the theorem you don’t even need any calculus. Since there are only two a’s in the game and only two b’s, everything depends on a single list of 16 probabilities adding up to +1. Because (in obvious notation) there only 16 possible values of the quadruple (A1, A2, B1, B2). As lambda varies it takes values in the set {-1, +1}^4. That’s a set of 16 elements. It’s easy to show that A1 B1 - A1 B2 - A2 B1 - A2 B2 always equals -2 or +2. Its mean value therefore lies between -2 and +2 (inclusive).

Bell wrote that Bohr would not have been impressed by Bell’s theorem. Bohr did not subscribe to local realism.

Re: Gull and Gill's theory

Post by FrediFizzx » Fri May 07, 2021 4:51 pm

Justo wrote:I am just curious. Bell is supposed to have shown that and do not exist such that

You say that is false. The question is how do you mathematically define the functions A and B that proves Bell is wrong?

I just posted it above your post or you can read Joy's papers. :roll:
.

Re: Gull and Gill's theory

Post by Justo » Fri May 07, 2021 4:49 pm

I am just curious. Bell is supposed to have shown that and do not exist such that

You say that is false. The question is how do you mathematically define the functions A and B that proves Bell is wrong?

Re: Gull and Gill's theory

Post by FrediFizzx » Fri May 07, 2021 4:26 pm

Here is how you get around the nonsense with the +/-1 substitution. You have to specify actual measurement functions for A and B. Note that the limit replacement functions drop out of the product calculation.

Image
Image

PlotArray1 is a partial list of the A and B outcome pairs. Note that the limit replacement function on the A and B functions give the 4 different outcome pairs.

Bell loses again! :mrgreen: :mrgreen: :mrgreen: :mrgreen:
.

Re: Gull and Gill's theory

Post by FrediFizzx » Wed May 05, 2021 2:05 am

gill1109 wrote:
FrediFizzx wrote:
gill1109 wrote:
FrediFizzx wrote:Actually this,



will have a range of -1 to +1 when the product is +1 or -1 with the high probability centered on zero instead of just being zero. Same with the ones in Gill's version of Gull's "proof". Well..., they do converge to zero at infinity.

As you should know Fred, it is easy to have P(a, b) take all values between -1 and +1 as a and b vary throughout their ranges. Bell gave the example of how to generate the triangle wave by a suitable choice of the set Lambda of all possible lambda, probability measure rho(lambda)d lambda on the set Lambda, and functions A and B with values in {-1, +1}.

Ah, he's back with more nonsense. I said, "when the product is +1 or -1...". The a and b vectors are out of the RHS of the equation so it doesn't matter what they are.



That is a legitimate substitution which produces nonsense. Without specifying actual functions for A and B, you end up with nonsense. Same with Gull's nonsense. And we already know that Bell's example is nonsense.
.

Sorry Fred, that was an illegitimate substitution which you just did. ...

How could it be illegitimate? Don't make nonsensical claims like that without backing it up. Are A and B equal to +1 or -1 or not? Of course they are. So their product is always going to be +1 or -1. The formulation produces nonsense just like I said. You have to figure out a way to keep the vectors a and b in the calculation while producing the +/-1's.
.

Re: Gull and Gill's theory

Post by gill1109 » Tue May 04, 2021 10:53 pm

FrediFizzx wrote:
gill1109 wrote:
FrediFizzx wrote:Actually this,



will have a range of -1 to +1 when the product is +1 or -1 with the high probability centered on zero instead of just being zero. Same with the ones in Gill's version of Gull's "proof". Well..., they do converge to zero at infinity.

As you should know Fred, it is easy to have P(a, b) take all values between -1 and +1 as a and b vary throughout their ranges. Bell gave the example of how to generate the triangle wave by a suitable choice of the set Lambda of all possible lambda, probability measure rho(lambda)d lambda on the set Lambda, and functions A and B with values in {-1, +1}.

Ah, he's back with more nonsense. I said, "when the product is +1 or -1...". The a and b vectors are out of the RHS of the equation so it doesn't matter what they are.



That is a legitimate substitution which produces nonsense. Without specifying actual functions for A and B, you end up with nonsense. Same with Gull's nonsense. And we already know that Bell's example is nonsense.
.

Sorry Fred, that was an illegitimate substitution which you just did.

Bell’s example is merely that: an example. Particular functions A and B, which happen to lead to the saw-tooth function (aka triangle wave) as correlation; not the negative cosine. What’s the problem with that? Try using Mathematica to check Bell’s maths. Or do an R Monte Carlo simulation. You’ll find that Bell’s maths is correct (the published papers not surprisingly do contain the odd typo, it’s always obvious how to fix them). Bell does not claim that this is a correct model from the point of view of physics when we are in the EPR-B situation. Obviously, it isn’t. That’s the point which Bell wanted to make.

Re: Gull and Gill's theory

Post by FrediFizzx » Sat Apr 24, 2021 10:49 am

gill1109 wrote:
FrediFizzx wrote:Actually this,



will have a range of -1 to +1 when the product is +1 or -1 with the high probability centered on zero instead of just being zero. Same with the ones in Gill's version of Gull's "proof". Well..., they do converge to zero at infinity.

As you should know Fred, it is easy to have P(a, b) take all values between -1 and +1 as a and b vary throughout their ranges. Bell gave the example of how to generate the triangle wave by a suitable choice of the set Lambda of all possible lambda, probability measure rho(lambda)d lambda on the set Lambda, and functions A and B with values in {-1, +1}.

Ah, he's back with more nonsense. I said, "when the product is +1 or -1...". The a and b vectors are out of the RHS of the equation so it doesn't matter what they are.



That is a legitimate substitution which produces nonsense. Without specifying actual functions for A and B, you end up with nonsense. Same with Gull's nonsense. And we already know that Bell's example is nonsense.
.

Top

CodeCogs - An Open Source Scientific Library